Top Banner
PART 1 B International Business Environment 94 Questions [1] Source: CMA 0676 1-34 Which of the following economic policies would not tend to correct a balance of payments deficit in the U.S.? A. Increase productivity in the manufacturing of U.S. exports. B. More effective use of monetary and fiscal policies to reduce inflation. C. A redistribution of economic aid and mutual defense burdens toward western European countries. D. Increase value of U.S. currency in relation to foreign currencies. [2] Source: CMA 0680 1-17 The value of the U.S. dollar in relation to other foreign currencies is A. Determined directly by the price of gold because the value of the U.S. dollar is tied to the price of gold. B. Set by the U.S. government in consultation with other foreign governments. C. Set along with the value of other currencies held by the International Monetary Fund. D. Determined by the forces of supply and demand on the foreign exchange markets. [3] Source: CMA 0680 1-20 When the U.S. dollar is expected to rise in value against foreign currencies, a U.S. company with foreign currency denominated receivables and payables should A. Slow down collections and speed up payments. B. Slow down collections and slow down payments. C. Speed up collections and speed up payments. D. Speed up collections and slow down payments. [4] Source: CMA 1281 1-12 If a country has only two factors of production, labor and capital, and it has a relative abundance of capital, the country will tend to A. Import capital-intensive goods and export labor-intensive goods. B. Refrain from trading owing to the abundance of capital. C. Export capital-intensive goods and import labor-intensive goods. D. Place an embargo on the export of capital-intensive goods. [5] Source: CMA 1281 1-16 In most recent years, the U.S. balance of payments has registered a deficit. This balance of payments deficit is a measure of the excess of A. Exports over imports. B. Imports over exports. C. Imports, private capital outflows, grants, and remittances over exports and private capital inflows. D. Goods imports over services imports. 1
49
Welcome message from author
This document is posted to help you gain knowledge. Please leave a comment to let me know what you think about it! Share it to your friends and learn new things together.
Transcript
Page 1: CMA Part 1B International Business Environment)

PART 1 BInternational Business Environment

94 Questions

[1] Source: CMA 0676 1-34 Which of the following economic policies would not tend to correct a balance of payments deficit in the U.S.?

A. Increase productivity in the manufacturing of U.S. exports.

B. More effective use of monetary and fiscal policies to reduce inflation.

C. A redistribution of economic aid and mutual defense burdens toward western European countries.

D. Increase value of U.S. currency in relation to foreign currencies.

[2] Source: CMA 0680 1-17 The value of the U.S. dollar in relation to other foreign currencies is

A. Determined directly by the price of gold because the value of the U.S. dollar is tied to the price of gold.

B. Set by the U.S. government in consultation with other foreign governments.

C. Set along with the value of other currencies held by the International Monetary Fund.

D. Determined by the forces of supply and demand on the foreign exchange markets.

[3] Source: CMA 0680 1-20 When the U.S. dollar is expected to rise in value against foreign currencies, a U.S. company with foreign currency denominated receivables and payables should

A. Slow down collections and speed up payments.

B. Slow down collections and slow down payments.

C. Speed up collections and speed up payments.

D. Speed up collections and slow down payments.

[4] Source: CMA 1281 1-12 If a country has only two factors of production, labor and capital, and it has a relative abundance of capital, the country will tend to

A. Import capital-intensive goods and export labor-intensive goods.

B. Refrain from trading owing to the abundance of capital.

C. Export capital-intensive goods and import labor-intensive goods.

D. Place an embargo on the export of

capital-intensive goods.

[5] Source: CMA 1281 1-16 In most recent years, the U.S. balance of payments has registered a deficit. This balance of payments deficit is a measure of the excess of

A. Exports over imports.

B. Imports over exports.

C. Imports, private capital outflows, grants, and remittances over exports and private capital inflows.

D. Goods imports over services imports.

[6] Source: CMA 0682 1-12 Of the following transactions, the one that would result in a debit entry in the U.S. balance of payments account is the

A. Receipt of dividends by an American corporation from its German subsidiary.

B. Buying of IBM shares by a Kuwaiti investor.

C. U.S. export of military equipment to Saudi Arabia.

D. Expenditure of a U.S. resident vacationing in France.

[7] Source: CMA 0682 1-13 Which one of the following transactions would result in a credit entry in the U.S. balance of payments account?

A. A New York bank pays $5,000 in interest to foreigners.

B. Volkswagen's U.S. subsidiary remits a dividend of $1 million to its parent company in Germany.

C. A U.S. exporter buys marine insurance from a British insurance company.

D. An Iowa farmer exports grain to Turkey.

[8] Source: CMA 1282 1-12 One may characterize the current international monetary system developed by the industrialized countries as a

A. Clean float. Freely floating exchange rates are determined solely by the forces of demand and supply.

B. Managed or dirty float. Central banks intervene in the foreign exchange market to influence the exchange rates.

C. Stable-rate system.

D. Gold-based system.

[9] Source: CMA 1282 1-13 An overvalued currency can be considered as

A. A tax on exports and a subsidy to imports.

1

Page 2: CMA Part 1B International Business Environment)

B. A tax on imports and a subsidy to exports.

C. A tax on both exports and imports.

D. A subsidy to both exports and imports.

[10] Source: CMA 1282 1-14 Given a spot rate of $1.8655 and a 90-day forward rate of $1.8723, the pound sterling in the forward market is

A. Being quoted at a premium.

B. Being quoted at a discount.

C. Undervalued.

D. Overvalued.

[11] Source: CMA 1282 1-17 Disregarding demand for its factors of production, a country's comparative advantage will lie in those goods whose production requires comparatively large amounts of its

A. Relatively scarce resources.

B. Relatively abundant resources.

C. Natural resources.

D. Capital.

[12] Source: CMA 1282 1-18 The difference between tariffs and quotas is

A. That the tariff is expressed as a percentage of price and the quota is expressed as an amount per unit.

B. That a tariff limits price and a quota limits quantities.

C. That a tariff is a tax and a quota is a subsidy.

D. That a tariff is a duty, whereas a quota is a limitation on quantities.

[13] Source: CMA 1282 1-19 A voluntary export quota is

A. A form of unilateral import quota.

B. Meaningless because of its voluntary nature.

C. A form of import quota negotiated with a country.

D. A violation of the World Trade Organization.

[14] Source: CMA 1285 1-25 The most significant advantage gained by a nation that participates in international trade is the

A. Higher rates of dividend and interest income received by its citizens and firms from their foreign

investments.

B. Greater availability of goods and services in domestic markets.

C. Enlarged revenues accruing to its national government from the imposition of import duties.

D. Increased profits and wages earned by firms and workers, respectively, in export industries.

[15] Source: CMA 1285 1-26 The appropriate remedy for the dumping of products by a foreign firm in the U.S. market would be to

A. Pass "buy American" laws.

B. Impose restrictions on U.S. exports to the offending country.

C. Impose countervailing duties.

D. Deny "most favored nation" treatment to exporters of the offending country.

[16] Source: CMA 1285 1-27 Trade restrictions such as tariffs and import quotas represent

A. An attempt by the government to bring about a more equitable distribution of income.

B. An increase in the unit costs of domestic producers who compete with foreign firms.

C. A subsidy paid by domestic consumers to foreign producers of the duty-burdened commodities.

D. A subsidy paid by domestic consumers to domestic producers of the duty-burdened commodities.

[17] Source: CMA 1285 1-28 The creation of a regional economic bloc of trading nations such as the European Union (EU),

A. Discourages foreign investment by nonmember multinational companies.

B. Encourages trade between the member nations and nonmember nations.

C. Requires the adoption of a common monetary unit.

D. Discriminates economically against nonmember nations.

[18] Source: CMA 1285 1-30 The dominant reason countries devalue their currencies is to

A. Improve the balance of payments.

B. Discourage exports without having to impose

2

Page 3: CMA Part 1B International Business Environment)

controls.

C. Curb inflation by increasing imports.

D. Slow what is regarded as too rapid an accumulation of international reserves.

[19] Source: CMA 1285 1-31 The U.S. balance of trade is decreased by

A. Foreign investments in the United States.

B. U.S. investments in foreign countries.

C. U.S. exports.

D. U.S. imports.

[20] Source: CMA 1285 1-32 Debt-servicing problems of less developed countries that primarily sell raw materials to the United States would be eased by

A. A recession in the United States with declines in interest rates.

B. An expanding U.S. economy with stable money supply growth.

C. An expansion of the lending authority of the World Bank.

D. A significant increase in the level of U.S. tariffs.

[21] Source: CMA 1285 1-33 The purchasing-power parity exchange rate

A. Is a fixed (pegged) exchange rate.

B. Is always equal to the market exchange rate.

C. Results in an undervalued currency of countries that are net importers.

D. Holds constant the relative price levels in two countries when measured in a common currency.

[22] Source: CMA 1285 1-34 An American importer of English clothing has contracted to pay an amount fixed in British pounds three months from now. If the importer worries that the U.S. dollar may depreciate sharply against the British pound in the interim, it would be well advised to

A. Buy pounds in the forward exchange market.

B. Sell pounds in the forward exchange market.

C. Buy dollars in the futures market.

D. Sell dollars in the futures market.

[23] Source: CMA 1285 1-35 The basic objective of the International Monetary Fund is to

A. Promote multilateral tariff reductions among nations.

B. Provide long-term loans to member countries that want to improve their infrastructure.

C. Provide a means of financing food imports so that more domestic resources can be shifted to industrialization.

D. Provide short-term loans to member nations experiencing temporary balance of payments difficulties.

[24] Source: CMA 0686 1-23 The economic reasoning dictating that each nation specialize in the production of goods that it produces relatively more efficiently than other nations and import those goods that are produced relatively more efficiently by other nations is called the doctrine of

A. Efficient trade.

B. Diminishing returns.

C. Relative competition.

D. Comparative advantage.

[25] Source: CMA 1286 1-16 The balance of trade is the

A. Same as the balance of the current account.

B. Balance on the capital account.

C. Balance on all international transactions.

D. Balance on the goods transactions in the current account.

[26] Source: CMA 1286 1-17 Special drawing rights (SDRs) are created by the International Monetary Fund (IMF) and

A. Are based on the value of a basket of five currencies and pegged to the value of gold.

B. Are based on the value of a basket of five currencies.

C. Are pegged to the price of gold.

D. Are a circulating currency like the European currency unit.

[27] Source: CMA 1286 1-18 If the annual U.S. inflation rate is expected to be 5% while the Italian lira is expected to depreciate against the U.S. dollar by 10%, an Italian firm importing from its U.S. parent can expect its lira costs for these imports to

A. Decrease by about 10%.

B. Decrease by about 5%.

3

Page 4: CMA Part 1B International Business Environment)

C. Increase by about 5%.

D. Increase by about 16.6%.

[28] Source: CMA 1286 1-19 An overvalued foreign currency exchange rate

A. Represents a tax on exports and a subsidy to

imports.

B. Represents a subsidy to exports and a tax on imports.

C. Has an effect on capital flows but no effect on trade flows.

D. Has no effect on capital flows but does affect trade flows.

[Fact Pattern #1]Suppose that 2 worker-hours are required to produce a clock radio in Japan, while 3 are required to do the same in Germany. In addition, 4 worker-hours are required to produce a television in Japan, while 5 are required to do the same in Germany.

[29] Source: CMA 1286 1-21 (Refers to Fact Pattern #1)Under conditions appropriate for free trade, both Japan and Germany would be better off if

A. Japan produces only televisions while Germany produces only clock radios and both use trade to meet the needs for the item not produced locally.

B. Japan produces only clock radios while Germany produces only televisions and both use trade to meet the needs of the item not produced locally.

C. Both produce some of each item and use trade to meet additional needs of a product.

D. Both Japan and Germany produce both products for internal use only with no need for international trade.

[30] Source: CMA 1286 1-22 (Refers to Fact Pattern #1)In the case of trade between Japan and Germany, the price of clock radios relative to the price of televisions is likely to be one clock radio for

A. 2.00 televisions.

B. 1.67 televisions.

C. 1.20 televisions.

D. 0.50 televisions.

[31] Source: CMA 1286 1-20 Given a spot exchange rate for the U.S. dollar against the pound sterling of 1.4925 and a 90-day forward rate of 1.4775

A. The pound sterling is at a discount against the dollar and undervalued in the forward market.

B. The pound sterling is at a premium against the dollar and overvalued in the forward market.

C. The forward pound sterling is at a discount against the dollar.

D. The forward pound sterling is at a premium against the dollar.

[32] Source: CMA 0687 1-21 The World Trade Organization

A. Introduced fixed exchange rates among the United States, Canada, and members of the European Union.

B. Created the International Monetary Fund.

C. Encourages reductions in trade barriers between countries.

D. Introduced exchange rates that adjust in response to changes in trade deficits and surpluses.

[33] Source: CMA 1287 1-24 In the modern world economy, balance-of-payments deficits and surpluses can be eliminated

A. Through the market mechanism of flexible exchange rates.

B. If all nations adopt tight monetary policies.

C. Only if trade between nations is curtailed.

D. When the opportunity costs of production are made the same in all countries.

[34] Source: CMA 1287 1-25 One of the major consequences of international trade between nations is

A. Higher prices for consumers.

B. A decreased variety of consumer products.

C. The possibility for total world output to increase.

D. Reduced competition for businesses.

[35] Source: CMA 1287 1-26 Which one of the following statements concerning international trade and protection is true?

A. Protection is necessary in order to keep U.S. money in the United States.

B. When two nations trade, one must gain while the other must lose.

C. The United States cannot compete with nations whose labor costs are lower.

4

Page 5: CMA Part 1B International Business Environment)

D. U.S. imports raise living standards in the United States.

[36] Source: CMA 1287 1-27 One consequence of the imposition of tariffs or quotas on imported products is

A. Lower prices for domestic products that compete with affected imports.

B. Domestic industry opposition to protection from imports.

C. Additional consumption of the affected imported products.

D. Higher prices for the affected imported products.

[37] Source: CMA 1287 1-28 If the value of the U.S. dollar in foreign currency markets changes from $1 = 6 marks to $1 = 4 marks,

A. The German mark has depreciated against the dollar.

B. German imported products in the U.S. will become more expensive.

C. U.S. tourists in Germany will find their dollars will buy more German products.

D. U.S. exports to Germany should decrease.

[38] Source: CMA 1287 1-29 If consumers in Japan decide they would like to increase their purchases of consumer products made in the United States, in foreign currency markets there will be a tendency for

A. The supply of dollars to increase.

B. The supply of dollars to decrease.

C. The Japanese yen to appreciate relative to the U.S. dollar.

D. The demand for dollars to increase.

[39] Source: CMA 1287 1-30 If the U.S. dollar declines in value relative to the currencies of many of the U.S. trading partners, the likely result is that

A. Foreign currencies will depreciate against the dollar.

B. The U.S. balance of payments deficit will become worse.

C. U.S. exports will tend to increase.

D. U.S. imports will tend to increase.

[40] Source: CMA 0688 1-21 The most widely used currency in international business

today is the

A. United States dollar.

B. Euro.

C. Japanese yen.

D. Swiss franc.

[41] Source: CMA 0688 1-22 Exchange rates are determined by

A. Each industrial country's government.

B. The International Monetary Fund.

C. Supply and demand in the foreign currency market.

D. Exporters and importers of manufactured goods.

[42] Source: CMA 0688 1-23 If risk is purposely undertaken in the foreign currency market, the investor in foreign currency then becomes

A. A speculator.

B. An arbitrageur.

C. Involved in hedging.

D. An exporter.

[43] Source: CMA 0688 1-24 Special drawing rights

A. Were created to reduce world inflation.

B. Represent the creation of new international liquidity by the International Monetary Fund.

C. Are a new currency used by commercial banks in the international market.

D. Are allocated to less developed countries every year by the International Monetary Fund.

[44] Source: CMA 0688 1-25 In foreign currency markets, the phrase "managed float" refers to the

A. Tendency for most currencies to depreciate in value.

B. Discretionary buying and selling of currencies by central banks.

C. Necessity of maintaining a highly liquid asset, such as gold, to conduct international trade.

D. Fact that actual exchange rates are set by private business people in trading nations.

[45] Source: CMA 0688 1-29

5

Page 6: CMA Part 1B International Business Environment)

In trade discussions between the United States and Japanese governments, if Japan voluntarily agrees to restrict automobile exports, which one of the following is true?

A. This restriction has no effect on the price paid by the consumer.

B. The United States government gains from these restrictions.

C. Profit margins for Japanese auto manufacturers have increased.

D. Under this agreement, the Japanese have an incentive to export less expensive cars to the United States.

[46] Source: CMA 0688 1-30 What is the role of gold in the present international monetary system?

A. Gold is quoted in United States dollars only.

B. All of the major currencies of the world, except the United States dollar, have a fixed value in terms of gold.

C. Gold is like any other asset whose value depends upon supply and demand.

D. Gold is the reserve asset of the International Monetary Fund.

[Fact Pattern #2]

One Unit of Resources Can Produce Soybeans (tons) Chips (units) --------------- ------------- Taiwan 6 1,500 United States 12 1,800

[47] Source: CMA 1288 1-12 (Refers to Fact Pattern #2)In trade between Taiwan and the United States,

A. Taiwan has an absolute advantage in producing soybeans.

B. The United States has a comparative advantage in producing soybeans.

C. Taiwan has a comparative advantage in producing soybeans.

D. The United States has a comparative advantage in producing chips.

[48] Source: CMA 1288 1-13 (Refers to Fact Pattern #2)If there were free trade between the two countries, which one of the following statements would be true?

A. Only the United States will gain from free trade.

B. The United States would specialize in the production of both chips and soybeans.

C. The United States will export chips to Taiwan.

D. Taiwan will specialize in the production of chips.

[49] Source: CMA 1288 1-14 (Refers to Fact Pattern #2)Assuming free trade between the United States and Taiwan, the relative prices of soybeans and chips will be

A. Exactly 1 ton of soybeans for 250 chips.

B. Between 150 to 250 chips for 1 ton of soybeans.

C. Between 1.2 to 2.0 tons of soybeans for 100 chips.

D. Exactly 1 ton of soybeans for 120 chips.

[50] Source: CMA 1288 1-15 The U.S. dollar has a free-floating exchange rate. When the dollar has fallen considerably in relation to other currencies, the

A. Trade account in the U.S. balance of payments is neither in a deficit nor in a surplus because of the floating exchange rates.

B. Capital account in the U.S. balance of payments is neither in a deficit nor in a surplus because of the floating exchange rates.

C. Fall in the dollar's value cannot be expected to have any effect on the U.S. trade balance.

D. Cheaper dollar helps U.S. exporters of domestically produced goods.

[51] Source: CMA 1288 1-16 One U.S. dollar is being quoted at 120 Japanese yen on the spot market and at 123 Japanese yen on the 90-day forward market; hence, the annual effect in the forward market is that the

A. U.S. dollar is at a premium of 10%.

B. U.S. dollar is at a premium of 2.5%.

C. U.S. dollar is at a discount of 10%.

D. U.S. dollar is at a premium of 0.025%.

[52] Source: CMA 1288 1-17 Caroline Brown, the product manager for a U.S. computer manufacturer, is being asked to quote prices of desktop computers to be used in Kuwait. The Kuwaiti government wants the price in British pounds, for delivery next year. Brown knows that the general price level in the United States will increase by 3%. Her banker forecasts that the British pound will depreciate about 5% this year with respect to the U.S. dollar. If Brown is able to quote 700 pounds for immediate delivery, the price that should be quoted for delivery to Kuwait next year is about

A. 735 pounds.

6

Page 7: CMA Part 1B International Business Environment)

B. 721 pounds.

C. 757 pounds.

D. 745 pounds.

[53] Source: CMA 1288 1-18 Consider a world consisting of only two countries, Canada and Italy. Inflation in Canada in 1 year was 5%, and in Italy 10%. Which one of the following statements about the Canadian exchange rate (rounded) during that year will be true?

A. Inflation has no effect on the exchange rates.

B. The Canadian dollar will appreciate by 5%.

C. The Canadian dollar will depreciate by 5%.

D. The Canadian dollar will depreciate by 15%.

[54] Source: CMA 1288 1-19 When analyzing a country's balance of payments accounts, the

A. "Current account" refers only to merchandise exports and imports.

B. "Current account" and "trade balance" are the same.

C. "Capital account" refers to the transactions related to the international movement of financial capital.

D. Country will be in financial jeopardy unless each component in the balance of payments accounts balances at the end of the year.

[55] Source: CMA 1288 1-20 Which one of the following statements about special drawing rights (SDRs) is correct?

A. Gold is used to determine the value of one SDR.

B. All of the major currencies of the world, except the U.S. dollar, have a fixed value in terms of SDRs.

C. SDRs were first introduced in 1969 by the International Monetary Fund to supplement existing reserves.

D. SDRs are the principal reserve asset of the U.S. government.

[56] Source: CMA 1288 1-21 The following transactions were noted for an economy whose currency is denominated in pesetas (Pta).

Amount in Pesetas ---------Imports of goods 20,300Exports of goods 15,760Domestic long-term investment in foreign countries 6,300Investment by foreigners in the country 1,400

Interest payments on foreign loans 3,700Gifts received from abroad 1,240When calculating the balance of trade for this economy, the

A. Current account has a surplus of Pta 7,000.

B. Capital account has a surplus of Pta 4,000.

C. Capital account has a deficit of Pta 7,700.

D. Current account has a deficit of Pta 7,000.

[57] Source: CMA 0691 1-16 Many domestic industries, such as cars and textiles, are partially protected from foreign competition by a system of import tariffs and import quotas. A major effect of such tariffs and quotas is to

A. Raise the domestic price of cars and textiles.

B. Lower the domestic price of cars and textiles.

C. Increase the volume of international trade in cars and textiles.

D. Reduce employment in the car and textile industries in the short run.

[58] Source: CMA 0691 1-18 Which one of the following items represents a credit in the U.S. balance of payment accounts?

A. U.S. imports of crude oil.

B. Expenditures of American tourists abroad.

C. Earnings belonging to foreign businesses that have U.S. plants.

D. Loans to Americans by foreigners.

[59] Source: CMA 1293 1-25 In relation to the balance of trade, all international transactions involving the purchase or sale of physical products between domestic and foreign countries are reflected in

A. The balance of the capital account.

B. Official reserves held by the central banks.

C. The official financing account.

D. The trade balance in the current account.

[60] Source: CMA 1293 1-26 Which one of the following did not contribute to the high value of the U.S. dollar during the 1980s?

A. Relatively high, real interest rates.

B. A large demand for U.S. dollars.

C. U.S. demand for foreign goods.

D. A stable U.S. government and currency.

7

Page 8: CMA Part 1B International Business Environment)

[61] Source: CMA 1293 1-27 The economic term used to describe the situation in which each nation specializes in the production of goods that it produces relatively more efficiently than other nations and imports those goods that are produced relatively more efficiently by other nations is called

A. Balance of trade.

B. Diminishing returns.

C. Relative competition.

D. Comparative advantage.

[62] Source: CIA 1193 IV-68 Which of the following provides the best justification for reducing trade barriers among nations?

A. The military self-sufficiency argument.

B. Diversification for stability argument.

C. The infant industry argument.

D. Increased total world output argument.

[63] Source: CIA 0594 IV-58 Which of the following statements does not properly describe a Eurodollar Deposit?

A. Eurodollar Deposits are U.S. dollar deposits in banks outside of the U.S.

B. Eurodollar Deposits are outside the direct control of the U.S. monetary authorities.

C. Eurodollar Deposit rates tend to be lower than domestic U.S. rates on equivalent instruments.

D. Interest rates on Eurodollar Deposits are tied to the London Interbank Offer Rate (LIBOR).

[64] Source: CIA 0592 IV-70 A short-term speculative rise in the world-wide value of domestic currency could be moderated by a central bank decision to

A. Sell domestic currency in the foreign exchange market.

B. Buy domestic currency in the foreign exchange market.

C. Sell foreign currency in the foreign exchange market.

D. Increase domestic interest rates.

[65] Source: CIA 1190 IV-58 A U.S. company and a German company purchased the same stock on the German stock exchange and held the stock for 1 year. The value of the German mark weakened

against the dollar over this period. Comparing the returns of the two companies, the United States company's return will be

A. Lower.

B. Higher.

C. The same.

D. Higher in the short-run but lower in the long-run.

[66] Source: CMA 0694 1-4 If the central bank of a country raises interest rates sharply, the country's currency will likely

A. Increase in relative value.

B. Remain unchanged in value.

C. Decrease in relative value.

D. Decrease sharply in value at first and then return to its initial value.

[67] Source: CMA 0694 1-5 All of the following are true about international trade except that

A. The gains from international trade depend on specialization with comparative advantage.

B. Absolute advantage without comparative advantage does not result in gains from international trade.

C. Absolute advantage is defined as the ability of one nation to produce a product at a relatively lower opportunity cost than another nation.

D. If there is reciprocal absolute advantage between two countries, specialization will make it possible to produce more of each product.

[68] Source: CMA 0695 1-23 Which one of the following groups would be the primary beneficiary of a tariff?

A. Domestic producers of export goods.

B. Domestic producers of goods protected by the tariff.

C. Domestic consumers of goods protected by the tariff.

D. Foreign producers of goods protected by the tariff.

[69] Source: CMA 0695 1-24 Assuming exchange rates are allowed to fluctuate freely, which one of the following factors would likely cause a nation's currency to appreciate on the foreign exchange market?

8

Page 9: CMA Part 1B International Business Environment)

A. A relatively rapid rate of growth in income that stimulates imports.

B. A high rate of inflation relative to other countries.

C. A slower rate of growth in income than in other countries, which causes imports to lag behind exports.

D. Domestic real interest rates that are lower than real interest rates abroad.

[70] Source: CIA 0595 IV-59 Interest rates received by depositors on Eurodollar deposits tend to be higher than domestic U.S. rates on equivalent instruments because

A. Borrowers pay higher rates than domestic U.S. rates on equivalent instruments.

B. The deposits involve different currencies.

C. Eurodollar deposits are for smaller amounts.

D. The Eurodollar market is outside the direct control of the U.S. monetary authorities and has lower costs.

[71] Source: CIA 1195 IV-66 Of the following, a characteristic of Eurobonds is that they are

A. Always denominated in Eurodollars.

B. Always sold in some country other than the one in whose currency the bond is denominated.

C. Sold outside the country of the borrower but are denominated in the currency of the country in which the issue is sold.

D. Generally issued as registered bonds.

[72] Source: CIA 0596 IV-69 If a country has a freely floating exchange rate system and is experiencing an appreciation in the external value of its

currency, it has

A. A current account deficit and a capital account surplus.

B. A current account surplus and a capital account deficit.

C. Shrinking official reserves.

D. No balance of payments surplus or deficit after short-run exchange-rate adjustments are complete.

[73] Source: CIA 1195 IV-67 Two countries have flexible exchange rate systems and an active trading relationship. If incomes [List A] in country 1, everything else being equal, then the currency of country 1 will tend to [List B] relative to the currency of country 2.

List A List B --------------- --------------- A.

Rise Remain constant B.

Fall Depreciate C.

Rise Depreciate D.

Remain constant Appreciate

[74] Source: CIA 1196 IV-64 A company has a foreign-currency-denominated trade payable, due in 60 days. In order to eliminate the foreign currency exchange-rate risk associated with the payable, the company could

A. Sell foreign currency forward today.

B. Wait 60 days and pay the invoice by purchasing foreign currency in the spot market at that time.

C. Buy foreign currency forward today.

D. Borrow foreign currency today, convert it to domestic currency on the spot market, and invest the funds in a domestic bank deposit until the invoice payment date.

[75] Source: CIA 1196 IV-73 If the exchange rate has changed from 1 U.S. dollar being worth 5 French francs to a rate of 1 U.S. dollar being worth 5.5 French francs,

A. The U.S. dollar has appreciated by 10%.

B. The U.S. dollar has depreciated by 10%.

C. The French franc has appreciated by 20%.

D. The French franc has depreciated by 20%.

[76] Source: CIA 1196 IV-74 Assuming that the real rate of interest is the same in both countries, if country A has a higher nominal interest rate than country B, the currency of country A will likely be selling at a

A. Forward discount relative to the currency of country B.

B. Forward premium relative to the currency of country B.

C. Spot discount relative to the currency of country B.

D. Spot premium relative to the currency of country B.

[77] Source: CIA 1195 IV-68 A direct effect of imposing a protective tariff on an item for

9

Page 10: CMA Part 1B International Business Environment)

which there are both foreign and domestic producers is that domestic producers will sell <List A> of the item while domestic consumers consume <List B> of the item.

List A List B ------ ------ A.

More More B.

More Less C.

Less More D.

Less Less

[78] Source: CIA 0595 IV-64 Which of the following measures create the most restrictive barrier to exporting to a country?

A. Tariffs.

B. Quotas.

C. Embargoes.

D. Exchange controls.

[79] Source: CIA 0594 IV-64 Which of the following is a tariff?

A. Licensing requirements.

B. Consumption taxes on imported goods.

C. Unreasonable standards pertaining to product quality and safety.

D. Domestic content rules.

[80] Source: CIA 0594 IV-65 Which of the following is a direct effect of imposing a protective tariff on an imported product?

A. Lower domestic prices on the imported item.

B. Lower domestic consumption of the item.

C. Reduced domestic production of the item.

D. Higher sales revenues for foreign producers of the item.

[81] Source: Publisher If the annual U.S. inflation rate is expected to be 3%, and the German mark is expected to depreciate against the U.S. dollar by 12%, a German firm importing from its U.S. parent can expect the costs of imports denominated in marks to

A. Decrease by about 12%.

B. Decrease by about 5%.

C. Increase by about 3%

D. Increase by about 17%.

[82] Source: Publisher Holland produces 90 million cases of soda and 20 million pounds of cheese. To increase production of cheese to 30 million pounds, it must sacrifice 30 million cases of soda. Iceland produces 65 million cases of soda and 65 million pounds of cheese, but to produce 75 million pounds of cheese, it must sacrifice 10 million cases of soda. It can be concluded that

A. Holland has an absolute advantage in both soda and cheese production.

B. Iceland has an absolute advantage in soda production, and Holland has absolute advantage in cheese production.

C. Iceland has an absolute advantage in both soda and cheese production.

D. Holland has a comparative advantage in soda production, and Iceland has an absolute advantage with respect to cheese production.

[83] Source: Publisher A U.S. company invested $100,000 in Canada for one year at 10%. The Canadian dollar was selling at a spot rate of $.65 when the investment was made and $.70 when the investment matured. What was the approximate yield on this investment?

A. 2.14%

B. 7.69%

C. 10.00%

D. 18.46%

[84] Source: CIA 1196 IV-78 (Refer to Figure 3.) The graph depicts the domestic supply of and demand for a product that is also sold in the domestic market by foreign producers. The domestic producers are protected by a tariff of the amount Pt minus Pw. Pt is the domestic price including the tariff, and Pw is the world price for the product. The effect of the tariff is to

A. Reduce the domestic price from OPw to OPt.

B. Reduce foreign sales in the domestic market from ac to bc.

C. Increase domestic production from Ob to Oc.

D. Increase domestic production from Oa to Ob.

[Fact Pattern #3]

U.S. production possibilities table ----------------------------------- Production Alternatives -----------------------

10

Page 11: CMA Part 1B International Business Environment)

Product A B C D E F -- -- -- -- -- -- Lard 0 4 8 12 16 20 Beef sides 40 32 24 16 8 0 Canada production possibilities table ------------------------------------- Production Alternatives -----------------------Product A B C D E F -- -- -- -- -- --Lard 0 3 6 9 12 15Beef sides 60 48 36 24 12 0

[85] Source: Publisher (Refers to Fact Pattern #3)If the U.S. and Canada engage in trade, the terms of trade will be

A. Between 2 and 4 beef sides for 1 unit of lard.

B. Between 1/3 and 1/2 units of lard for 1 beef side.

C. Between 3 and 4 units of lard for 1 beef side.

D. Between 2 and 4 units of lard for 1 beef side.

[86] Source: Publisher (Refers to Fact Pattern #3)Assume that, prior to specialization and trade, the U.S. and Canada both choose production possibility C. If each specializes according to its comparative advantage, the resulting gains from specialization and trade will be

A. 6 units of lard.

B. 8 units of lard.

C. 6 units of lard and 8 beef sides.

D. 8 units of lard and 6 beef sides.

[87] Source: Publisher (Refers to Fact Pattern #3)Each nation produces only one product in accordance with its comparative advantage, and the terms of trade are set at 3 beef sides for 1 unit of lard. In this case, the U.S. can obtain a maximum combination of 8 units of lard and

A. 12 beef sides.

B. 24 beef sides.

C. 36 beef sides.

D. 48 beef sides.

[Fact Pattern #4]

Bulgaria production possibilities table --------------------------------------- Production Alternatives --------------------------------Product A B C D E F ----- ----- --- --- --- ---Cars 1,500 1,200 900 600 300 0Tractors 0 100 200 300 400 500 Andorra production possibilities table

-------------------------------------- Production Alternatives -------------------------------Product A B C D E ----- ----- ----- ----- ---Cars 4,000 3,000 2,000 1,000 0Tractors 0 200 400 600 800

[88] Source: Publisher (Refers to Fact Pattern #4)In Bulgaria, the comparative cost of

A. 1 car is 3 tractors.

B. 1 tractor is 1/3 car.

C. 1 car is 1/3 tractor.

D. 3 tractors is 1 car.

[89] Source: Publisher (Refers to Fact Pattern #4)Which of the following statements is not true?

A. Bulgaria should specialize in the production of tractors.

B. Bulgaria has a comparative advantage in the production of tractors.

C. Andorra should specialize in the production of tractors.

D. Andorra has a comparative advantage in the production of cars.

[90] Source: Publisher (Refers to Fact Pattern #4)The terms of trade will be

A. Greater than 7 cars for 1 tractor.

B. Between 7 cars for 1 tractor and 5 cars for 1 tractor.

C. Between 5 cars for 1 tractor and 3 cars for 1 tractor.

D. Less than 3 cars for 1 tractor.

[91] Source: Publisher (Refers to Fact Pattern #4)Assume that, if Bulgaria does not specialize, it will produce alternative C and that, if Andorra does not specialize, it will select alternative B. The gains from specialization will be

A. 100 cars and 100 tractors.

B. 200 cars and 200 tractors.

C. 400 cars and 500 tractors.

D. 500 cars and 500 tractors.

11

Page 12: CMA Part 1B International Business Environment)

[92] Source: Publisher If the U.S. dollar-peso exchange rate is $1 for 9 pesos, a product priced at 45 pesos will cost a U.S. consumer

A. $0.20

B. $5

C. $45

D. $405

[93] Source: Publisher If a U.S. firm can buy 」20,000 for $100,000, the rate of exchange for the pound is

A. $.20

B. $5

C. $20

D. $50

[94] Source: Publisher The spot rate of the French franc is $.90. If the spot rate one year from now is $.85, the franc will have

A. Appreciated by 5.56%.

B. Depreciated by 5.56%.

C. Appreciated by 5.88%.

D. Depreciated by 5.88%.

12

Page 13: CMA Part 1B International Business Environment)

CMA PART 1 BInternational Business Environment

ANSWERS

[1] Source: CMA 0676 1-34

Answer (A) is incorrect because increasing productivity will lower the price of U.S. exports which will increase the amount of exports demanded, and thus reduce the balance of payments deficit.

Answer (B) is incorrect because as the rate of inflation slows down, prices charged to overseas buyers are reduced, which will increase exports (and reduce the balance of payments deficit).

Answer (C) is incorrect because if more money were given to countries the U.S. trades with, they would buy more U.S. exports, thus correcting a balance of payments deficit.

Answer (D) is correct. A balance of payments deficit exists when the fixed or managed exchange rate is too high. "Too high" is when the fixed price is higher than the equilibrium price would be if market forces were at work. To correct a balance of payments deficit the price of dollars must decrease or other means must be undertaken to raise the real value to the fixed level. If the value of U.S. currency is increased, the deficit will grow.

[2] Source: CMA 0680 1-17

Answer (A) is incorrect because the value of the dollar is not formally tied to gold. While there may be a long-term relationship between gold and the value of the dollar, there are often inverse (or random) short-term fluctuations.

Answer (B) is incorrect because an exchange rate set by the government is called a fixed exchange rate. The old international monetary system which used fixed exchange rates collapsed because of its inefficient handling of currency prices.

Answer (C) is incorrect because the International Monetary Fund has little effect on the valuation of currencies.

Answer (D) is correct. Exchange rates are determined by the forces of supply and demand on the exchange markets. Often other forces try to intervene in this process of exchange rate determination, but these reflect only short-run policies. An example of this type of policy would be government or central bank intervention in the international money markets.

[3] Source: CMA 0680 1-20

Answer (A) is incorrect because the U.S. company should expedite collections and defer payments.

Answer (B) is incorrect because the U.S. company

should expedite collections and defer payments.

Answer (C) is incorrect because the U.S. company should expedite collections and defer payments.

Answer (D) is correct. The proper action would be to increase collections and decrease payments. Collections should be made quickly and converted into dollars to sustain the increase in their value as the dollar appreciates. Decreasing payments would be profitable because, as the company exchanges dollars for foreign currency at a later date, it will receive more of the foreign currency, thus lowering its real cost.

[4] Source: CMA 1281 1-12

Answer (A) is incorrect because a country will import capital-intensive goods and export labor-intensive goods only if it has relatively abundant labor.

Answer (B) is incorrect because trade could increase the welfare of the country.

Answer (C) is correct. A country that has relatively abundant capital will have a comparative advantage in capital-intensive goods. Accordingly, these goods should be exported. If the country has less abundant labor, it should import labor-intensive goods because it will be a less efficient producer of such.

Answer (D) is incorrect because an embargo on the export of capital-intensive goods is economically

irrational if the country has a comparative advantage in their production.

[5] Source: CMA 1281 1-16

Answer (A) is incorrect because it excludes capital inflows and outflows, grants, and remittances.

Answer (B) is incorrect because it excludes capital inflows and outflows, grants, and remittances.

Answer (C) is correct. The balance of payments is defined as the excess of imports, private capital outflows, grants, and remittances over exports and private capital inflows. When there is a surplus in the balance of payments, more domestic goods may have been sold abroad than were imported, and/or foreigners may have invested more capital in the domestic country than domestic citizens invested abroad. For this reason, a surplus is considered a favorable balance of payments. Just the opposite is true for a deficit in the balance of payments.

Answer (D) is incorrect because it excludes exports completely.

[6] Source: CMA 0682 1-12

Answer (A) is incorrect because it increases the demand for U.S. currency.

Answer (B) is incorrect because it represents an increase in demand for U.S. assets by foreigners.

13

Page 14: CMA Part 1B International Business Environment)

Answer (C) is incorrect because it reflects an increase in U.S. goods and services demanded by foreigners.

Answer (D) is correct. Since debits represent unfavorable items, expenditures by U.S. residents vacationing abroad require debits. Buying French goods and services means using dollars to acquire francs, thus increasing the demand for French currency.

[7] Source: CMA 0682 1-13

Answer (A) is incorrect because it reflects an increase in demand for foreign currency by

Americans.

Answer (B) is incorrect because it is a private capital outflow and would appear as a debit in the balance of payments account.

Answer (C) is incorrect because it represents imports by the U.S. of insurance services.

Answer (D) is correct. Exports help a country's balance of payments and are therefore considered a credit in the U.S. balance of payments account. Therefore, the Iowa farmer's export of grain to Turkey would be recorded as a credit.

[8] Source: CMA 1282 1-12

Answer (A) is incorrect because a clean float system does not exist owing to central bank intervention.

Answer (B) is correct. Today's international monetary system usually permits exchange rates to float freely. However, central banks occasionally intervene to avoid large fluctuations. Accordingly, the system is called a managed or dirty float system.

Answer (C) is incorrect because, as a result of the floating exchange rate system, rates are not stable.

Answer (D) is incorrect because the gold-based system was abandoned in 1973.

[9] Source: CMA 1282 1-13

Answer (A) is correct. If a currency is overvalued relative to a foreign currency, it will purchase more of that currency, thereby encouraging imports of foreign goods. Similarly, exports are discouraged because the foreign currency will purchase less of the overvalued currency. Therefore, an overvalued currency operates as a tax on exports and a subsidy to imports.

Answer (B) is incorrect because it describes the effect of an undervalued currency.

Answer (C) is incorrect because overvaluation does not operate as a tax on imports.

Answer (D) is incorrect because overvaluation does

not operate as a subsidy to exports.

[10] Source: CMA 1282 1-14

Answer (A) is correct. Given a spot rate value of $1.8655 and a forward rate of $1.8723, the value of the pound is greater in the futures market than in the spot market. Accordingly, the price is being quoted at a premium because speculators expect the currency to appreciate.

Answer (B) is incorrect because the price is quoted at a discount if the spot rate is higher than the forward rate.

Answer (C) is incorrect because undervaluation refers to comparisons of currencies.

Answer (D) is incorrect because overvaluation refers to comparisons of currencies.

[11] Source: CMA 1282 1-17

Answer (A) is incorrect because scarcity causes a higher opportunity cost.

Answer (B) is correct. If one good requires comparatively large amounts of a resource, a country with a relatively large amount of that resource should have the advantage in the production of that good.

Answer (C) is incorrect because the country may have relatively less abundant natural resources.

Answer (D) is incorrect because the country may have relatively less abundant capital.

[12] Source: CMA 1282 1-18

Answer (A) is incorrect because a quota is a restriction on a quantity of units.

Answer (B) is incorrect because a tariff is a per unit tax.

Answer (C) is incorrect because a quota is a restriction on quantity, not a subsidy.

Answer (D) is correct. A tariff is a duty imposed on imports. A quota is a restriction on the quantity of a good which may be imported.

[13] Source: CMA 1282 1-19

Answer (A) is incorrect because a unilateral import quota is imposed by the importing country on the exporting country without agreement with the other country.

Answer (B) is incorrect because, if the quota is adhered to, it will be quite meaningful.

Answer (C) is correct. A voluntary export quota is in reality a form of negotiated import quota from the perspective of the other country involved.

14

Page 15: CMA Part 1B International Business Environment)

Answer (D) is incorrect because of the voluntary nature of the quota.

[14] Source: CMA 1285 1-25

Answer (A) is incorrect because it concerns capital movements, not trade. Also, excessive capital outflows produce a negative effect on the balance of payments.

Answer (B) is correct. When a nation engages in international trade, it mostly trades goods and services in which it has a comparative advantage for goods and services of which it is a relatively less efficient producer. A nation may thus concentrate on production of those goods for which it has a comparative advantage. Consequently, imported products are sold for real prices lower than if they were produced in the importing nation. The result is greater availability of goods and services in the domestic market.

Answer (C) is incorrect because import duties are charged to domestic consumers and thus result in no increase in national resources.

Answer (D) is incorrect because the export industries may well account for only a small fraction of a nation's activity, so the greater availability of goods and services is a more significant effect of international trade than the benefits to exporters.

[15] Source: CMA 1285 1-26

Answer (A) is incorrect because the passing of "buy American" laws could result in a decline in overall domestic consumption, higher prices, and retaliatory foreign action.

Answer (B) is incorrect because a country does not benefit from restricting its exports.

Answer (C) is correct. Dumping is the practice of supporting exports by selling products at a lower price in foreign markets than in the domestic market. The result is that foreign goods (such as certain items produced in the Far East) can be purchased in the U.S. at a price much lower than would be charged by a U.S. manufacturer. Since dumping lowers the price of foreign goods, the appropriate remedy would be for the importing nation to impose a tariff that would reduce the price differential.

Answer (D) is incorrect because denying "most favored nation" treatment would make trade with a country more difficult and would be a more extreme remedy than necessary.

[16] Source: CMA 1285 1-27

Answer (A) is incorrect because the government is not trying to equitably distribute income. Import restrictions are either politically expedient, or intended to support critical industries that might be needed in time of war.

Answer (B) is incorrect because restrictions do not

increase manufacturing costs; they only permit those with higher costs to compete effectively.

Answer (C) is incorrect because the benefit of the subsidy accrues to the inefficient domestic producer.

Answer (D) is correct. Trade restrictions are designed to protect domestic industries that cannot effectively meet foreign competition. Tariffs and quotas therefore cause consumers to pay higher prices and to consume fewer goods and services. In effect, consumers pay a subsidy to domestic producers. The long-term results are a reduction in trade and misallocation of resources to less efficient industries.

[17] Source: CMA 1285 1-28

Answer (A) is incorrect because foreign investment may be welcomed under the appropriate conditions.

Answer (B) is incorrect because the EU provides incentives to trade with other members, not nonmembers.

Answer (C) is incorrect because currencies are not affected by bloc membership.

Answer (D) is correct. A trading bloc provides trading incentives to member nations and discriminates against nonmember nations. For example, the European Union calls for abolition of internal tariffs and import quotas, free movement of capital and labor within the market, and implementation of common policies for the member nations. However, the EU also imposed a common system of tariffs on goods of nonmember nations.

[18] Source: CMA 1285 1-30

Answer (A) is correct. Currency devaluations result in a change in the balance of payments. A devaluation means that other currencies will buy more of the devaluing nation's currency, and the prices of goods denominated in the devalued currency are therefore cheaper. A devaluation usually results in an increase in exports, a decrease in imports (caused by higher relative input prices), and an improved balance of trade.

Answer (B) is incorrect because a devaluation will encourage exports.

Answer (C) is incorrect because a devaluation will discourage imports and may encourage domestic inflation. Foreign goods will be more expensive.

Answer (D) is incorrect because devaluation most likely occurs when a country is losing its reserves.

[19] Source: CMA 1285 1-31

Answer (A) is incorrect because foreign investments in the United States is a factor in the balance of payments but not trade.

Answer (B) is incorrect because U.S. investments in

15

Page 16: CMA Part 1B International Business Environment)

foreign countries is a factor in the balance of payments but not trade.

Answer (C) is incorrect because exports increase the balance of trade.

Answer (D) is correct. The balance of payments embraces all payments made by one nation to another, including capital movements. The balance of trade is the difference between imports and exports of goods and services over a given period. In T-account form, exports are credits and imports debits. Assuming that a credit balance reflects a positive balance of trade, imports will decrease a positive balance while exports will increase it.

[20] Source: CMA 1285 1-32

Answer (A) is incorrect because a recession would result in less U.S. demand for raw materials from abroad and a reduction in funds available to the underdeveloped nations to pay debts.

Answer (B) is correct. An expanding U.S. economy would result in greater demand for raw materials from these countries. Also, since the money supply and interest rates are inversely proportional (when the money supply is rising, interest rates are falling), less developed nations could borrow again at lower rates. Moreover, if the money supply is rising, inflation might increase and U.S. dollars would become cheaper, thereby easing the burden of foreign debtors with obligations payable in dollars.

Answer (C) is incorrect because an expansion of lending authority could only increase the debt outstanding and make it less possible for less developed countries to service their debts.

Answer (D) is incorrect because tariffs would reduce exports to the U.S. and thus the funds available for debt service.

[21] Source: CMA 1285 1-33

Answer (A) is incorrect because purchasing-power parity is achieved through floating exchange rates.

Answer (B) is incorrect because the purchasing-power parity exchange rate is a long-run measure, but the market rate may reflect short-term or medium-term conditions.

Answer (C) is incorrect because purchasing-power parity does not affect the valuation of currency.

Answer (D) is correct. The purchasing-power parity theorem states that, in the long run, the real price of a good in country A will equal the price of the same good in country B when the prices are expressed in a common currency and converted at the current exchange rate (adjustments for tariffs, taxes, or transportation cost may need to be made).

[22] Source: CMA 1285 1-34

Answer (A) is correct. The American importer

should buy pounds now. If the dollar depreciates against the pound in the next 90 days, the gain on the forward exchange contract would offset the loss from having to pay more dollars to satisfy the liability.

Answer (B) is incorrect because selling pounds would compound the risk of loss for someone who has incurred a liability. However, it would be an appropriate hedge of a receivable denominated in pounds.

Answer (C) is incorrect because the importer needs pounds, not dollars.

Answer (D) is incorrect because, although buying pounds might be equivalent to selling dollars for pounds, this is not the best answer. This choice does not state what is received for the dollars.

[23] Source: CMA 1285 1-35

Answer (A) is incorrect because the World Trade Organization (WTO) is an attempt by the signatory nations to reduce tariffs and import quotas.

Answer (B) is incorrect because the World Bank provides credits for development purposes.

Answer (C) is incorrect because the U.S. Food for Peace program permits other countries to purchase U.S. farm products with other currencies.

Answer (D) is correct. The resources of the IMF consist of a pool of currency from which short-term loans can be made to member nations who are experiencing temporary balance of payments problems. Special drawing rights (SDRs) have also been created on the books of IMF to serve as an additional reserve for member nations to use in settling accounts.

[24] Source: CMA 0686 1-23

Answer (A) is incorrect because efficient trade is not meaningful in this context.

Answer (B) is incorrect because diminishing returns is not meaningful in this context.

Answer (C) is incorrect because relative competition is not meaningful in this context.

Answer (D) is correct. The doctrine of comparative advantage relates to comparative costs within one country. It holds that a country should produce those products in which it has a comparative advantage, not necessarily those products in which it has an absolute advantage. The doctrine suggests that a country should produce those products for which the greatest efficiencies are attainable even if it could also produce other goods more efficiently than another nation. In the long run, importing a product in which a country has an absolute advantage but not a comparative advantage will result in an overall increase in global production.

[25] Source: CMA 1286 1-16

16

Page 17: CMA Part 1B International Business Environment)

Answer (A) is incorrect because the balance in the current account contains financial balances, but the balance of trade is concerned only with the balance of transactions in goods and services.

Answer (B) is incorrect because the capital account records net capital movements, not the balance of trade.

Answer (C) is incorrect because the balance of all international transactions includes items that are not goods or services, such as net capital movements, government transactions, and remittances.

Answer (D) is correct. The balance of payments embraces all payments made by one nation to another, including capital movements. The balance of trade is the difference between imports and exports of goods and services over a given period. In T-account form, exports are credits and imports debits. Assuming that a credit balance reflects a positive balance of trade, imports will decrease a positive balance while exports will increase it.

[26] Source: CMA 1286 1-17

Answer (A) is incorrect because in 1971 convertibility of gold into dollars was ended by the U.S. government. This effectively destroyed the international system of fixed exchange rates. SDRs serve the same function as gold once did in settling accounts between nations. Their value is based on a weighted average of currencies, not on the gold price.

Answer (B) is correct. Special drawing rights (SDRs) have been created on the books of the IMF by agreement of the IMF members to serve as an additional reserve for member nations to use in settling accounts. They are based on the weighted average value of several currencies.

Answer (C) is incorrect because in 1971 convertibility of gold into dollars was ended by the U.S. government. This effectively destroyed the international system of fixed exchange rates. SDRs serve the same function as gold once did in settling accounts between nations. Their value is based on a weighted average of currencies, not on the gold price.

Answer (D) is incorrect because SDRs are bookkeeping entries on the IMF books awarded in proportion to members' contribution quotas, not to borrowing rights.

[27] Source: CMA 1286 1-18

Answer (A) is incorrect because the combined effect of U.S. inflation and the decline in value of the lira would cause the lira costs for U.S. imports to increase.

Answer (B) is incorrect because the combined effect of U.S. inflation and the decline in value of the lira would cause the lira costs for U.S. imports to increase.

Answer (C) is incorrect because 5% is the difference between the currency depreciation and the inflation rate.

Answer (D) is correct. Assuming the original exchange rate is $1 to 2,000 lira and that U.S. inflation is 5%, the cost in lira to purchase what once cost $1 will now be 2,100 lira (2,000 x 1.05). However, if the lira also depreciates by 10%, that is, if the lira is expected to be worth 90% of its current value against the dollar, the exchange rate before inflation will be $1 to 2,222 lira (2,000 ・.9). At this rate, 2,333 lira (2,222 x 1.05) will be required to purchase $1.05. Lira costs will thus increase by over 16.6% (333 ・2,000).

[28] Source: CMA 1286 1-19

Answer (A) is correct. An overvalued exchange rate is a tax on exports because they will be overvalued in terms of the foreign currency. For example, if the true value of $1 is 5 marks but the exchange rate is $1 to 6 marks, the cost of goods priced in dollars will include a 20% tax for holders of marks. It is also a subsidy to imports because the overly high exchange rate causes the price of foreign goods and services to be undervalued. In the same example, $1 will buy 6 marks' worth of imports instead of 5, a 20% subsidy.

Answer (B) is incorrect because devaluation taxes imports and subsidizes exports.

Answer (C) is incorrect because both will be affected.

Answer (D) is incorrect because both will be affected.

[29] Source: CMA 1286 1-21

Answer (A) is incorrect because Japan and Germany should specialize in making radios and TV sets, respectively, and trade for what they do not produce.

Answer (B) is correct. Japan has a comparative advantage in radio production because its cost of producing radios is a smaller fraction of its cost of producing TV sets (2/4 = .5) than is true for Germany (its fraction is 3/5 = .6). Germany has a comparative advantage with regard to TV set production because its costs of producing TV sets (5/3 = 1.67) is a smaller fraction of its cost of producing radios than is true for Japan (its fraction is 4/2 = 2). Under the theory of comparative advantage, Japan should manufacture radios, Germany should make TV sets, and both should trade for what they do not produce.

Answer (C) is incorrect because Japan and Germany should specialize in making radios and TV sets, respectively, and trade for what they do not produce.

Answer (D) is incorrect because Japan and Germany should specialize in making radios and TV sets, respectively, and trade for what they do not produce.

[30] Source: CMA 1286 1-22

17

Page 18: CMA Part 1B International Business Environment)

Answer (A) is incorrect because 2.00 televisions is the inverse of the correct relationship.

Answer (B) is incorrect because Japan has an absolute advantage for both products. Hence, the correct relationship is 2 hours of production time for a radio and 4 hours for a television.

Answer (C) is incorrect because Japan has an absolute advantage for both products. Hence, the correct relationship is 2 hours of production time for a radio and 4 hours for a television.

Answer (D) is correct. Because Japan has an absolute advantage for both products, the relationship between Japanese radios and Japanese televisions would determine the price of each. Radios require 2 hours of production time and televisions require 4 hours, so the price of a radio is likely to be .50 televisions.

[31] Source: CMA 1286 1-20

Answer (A) is incorrect because there is no way to tell if the rates are over- or undervalued.

Answer (B) is incorrect because the pound is selling at a discount, not a premium.

Answer (C) is correct. A spot rate is defined as the exchange rate paid for delivery of currency "on the spot," i.e., today. A forward exchange rate is the future price of the currency. If the forward rate is greater than the spot rate, the currency is selling at a premium. If the forward rate is less than the spot rate, the currency is selling at a discount.

Answer (D) is incorrect because the pound is selling at a discount, not a premium.

[32] Source: CMA 0687 1-21

Answer (A) is incorrect because the WTO is a worldwide agreement concerning trade barriers, not exchange rates. Today, moreover, exchange rates are not pegged (fixed) but are allowed to float.

Answer (B) is incorrect because the IMF was founded in 1944 to stabilize exchange rates.

Answer (C) is correct. International trade agreements provide regulatory authority for businesses in international trade. The WTO, which was established on January 1, 1995, is the product of the Uruguay Round of international trade negotiations. It is a permanent body with a secretariat based in Geneva, Switzerland The WTO Agreement is a permanent set of commitments by more than 120 nations designed to prohibit trade discrimination among member nations and between imported and domestic products.

Answer (D) is incorrect because the WTO is a worldwide agreement concerning trade barriers, not exchange rates. Today, moreover, exchange rates are not pegged (fixed) but are allowed to float.

[33] Source: CMA 1287 1-24

Answer (A) is correct. If exchange rates are allowed to fluctuate, the value of a particular currency will be determined in accordance with the supply of and demand for that currency. For example, if U.S. exports to Japan are greater than imports, the dollar will be in great demand; thus, the dollar will be driven up in price relative to the Japanese yen. This increase in price will discourage the Japanese from buying U.S. goods. The decrease in Japanese purchases will then result, in principle, in less demand for the dollar and a movement back toward equilibrium in the export/import ratio.

Answer (B) is incorrect because tight money policies would not affect the balance of payments but would reduce economic activity generally.

Answer (C) is incorrect because deficits can be overcome by less drastic and counterproductive methods than eliminating trade.

Answer (D) is incorrect because demand for particular products is as important as relative costs in creating trade deficits.

[34] Source: CMA 1287 1-25

Answer (A) is incorrect because prices will be lower and greater quantities will be available with international trade.

Answer (B) is incorrect because variety will be increased by international trade.

Answer (C) is correct. Under the concept of comparative advantage, total world output will be maximized when each nation specializes in the products in which it has the lowest opportunity costs, that is, a comparative advantage. When nations specialize in what they produce most efficiently and then exchange with others, more is produced and consumed than if each nation tries to be self-sufficient. Specialization of labor is beneficial for individuals; the same principle applies to nations.

Answer (D) is incorrect because competition will be increased when more producers are in the market.

[35] Source: CMA 1287 1-26

Answer (A) is incorrect because U.S. investment abroad has proven to be beneficial.

Answer (B) is incorrect because both nations should benefit from international trade. Under the concept of comparative advantage, no country will be worse off through international trade.

Answer (C) is incorrect because the U.S. has other advantages not possessed by countries with low labor costs.

Answer (D) is correct. Imports can raise the standard of living because more goods are available to

18

Page 19: CMA Part 1B International Business Environment)

consumers. Tariffs, quotas, and other trade restraints are undesirable because free trade will maximize the total benefit to consumers worldwide (under the concept of comparative advantage).

[36] Source: CMA 1287 1-27

Answer (A) is incorrect because trade restricts full inflation since incentives to charge lower prices for domestic products are removed.

Answer (B) is incorrect because domestic industries support protection from imports to reduce competition.

Answer (C) is incorrect because consumption is reduced owing to higher prices and reduced supply.

Answer (D) is correct. Tariffs lead to higher prices on imported products. Similarly, the imposition of quotas leads to higher prices through an artificial limitation on supply.

[37] Source: CMA 1287 1-28

Answer (A) is incorrect because the mark has appreciated (increased in value) relative to the dollar.

Answer (B) is correct. The dollar has declined in value relative to the mark. If an American had previously wished to purchase a German product that was priced at 12 marks, the dollar price would have been $2. After the decline in value, the dollar cost of the item has increased to $3. Therefore, imports from Germany should decrease and exports increase.

Answer (C) is incorrect because dollars will buy fewer German products.

Answer (D) is incorrect because U.S. exports should increase. For example, if a U.S. product previously cost $3, the price to Germans would have been 18 marks. With the decline of the dollar, the Germans can now buy the $3 item for 12 marks.

[38] Source: CMA 1287 1-29

Answer (A) is incorrect because the demand for dollars, not the supply, will be affected by the decision to purchase additional U.S. products.

Answer (B) is incorrect because the demand for dollars, not the supply, will be affected by the decision to purchase additional U.S. products.

Answer (C) is incorrect because the dollar should appreciate relative to the yen owing to the increased demand for dollars.

Answer (D) is correct. The increase in demand for U.S. products will increase the demand for the dollars necessary to pay for those products.

[39] Source: CMA 1287 1-30

Answer (A) is incorrect because the dollar has

depreciated against foreign currencies.

Answer (B) is incorrect because the U.S. trade balance of payments should improve.

Answer (C) is correct. The decline in the value of the dollar reduces the prices of U.S. goods to foreigners and should increase exports. Also, foreign goods will be higher priced (in dollars) and imports from foreign countries should decrease, thus helping the U.S. balance of payments.

Answer (D) is incorrect because U.S. imports will decline. Foreign goods will be higher priced than before.

[40] Source: CMA 0688 1-21

Answer (A) is correct. The U.S. dollar is the most widely used currency in international markets today. It is considered much more stable than any of the third-world currencies. Thus, many third-world countries rely on the U.S. dollar for foreign trade.

Answer (B) is incorrect because the Euro is not as heavily used as the U.S. dollar.

Answer (C) is incorrect because the Japanese yen is not as heavily used as the U.S. dollar.

Answer (D) is incorrect because the Swiss franc is not as heavily used as the U.S. dollar.

[41] Source: CMA 0688 1-22

Answer (A) is incorrect because governments have only temporary influence, if any, on the setting of exchange rates.

Answer (B) is incorrect because the International Monetary Fund has only temporary influence, if any, on the setting of exchange rates.

Answer (C) is correct. Although currencies can be supported by various means for short periods, the primary determinant of exchange rates is the supply of and demand for the various currencies. Under current international agreements, exchange rates are allowed to "float." During periods of extreme fluctuations, however, governments and control banks may intervene to maintain stability in the market.

Answer (D) is incorrect because they have only temporary influence, if any, on the setting of exchange rates.

[42] Source: CMA 0688 1-23

Answer (A) is correct. An individual who purposely accepts exchange rate risk is a speculator. Speculators buy and sell foreign currencies in anticipation of favorable changes in rates.

Answer (B) is incorrect because an arbitrageur is someone who simultaneously buys foreign currency in one market and sells in another market at a slightly

19

Page 20: CMA Part 1B International Business Environment)

higher price. Thus, the arbitrageur's risk is slight.

Answer (C) is incorrect because hedging avoids the risk of foreign currency transactions for those who do not seek to gain from fluctuations in exchange rates. Hedging is the sale or purchase of a forward exchange contract to offset a possible exchange rate loss. When a forward exchange contract is intended and is effective as an economic hedge against an exposed net asset or net liability position (e.g., an outstanding receivable or liability denominated in a foreign currency), any exchange gain or loss on the forward contract will offset any exchange gain or loss on the exposed net asset or net liability position. Thus, no exchange gain or loss will result.

Answer (D) is incorrect because exporters and importers are likely to engage in hedging to avoid exchange rate risk.

[43] Source: CMA 0688 1-24

Answer (A) is incorrect because SDRs have nothing to do with inflation.

Answer (B) is correct. Special drawing rights (SDRs) were created by the International Monetary Fund (IMF) to enable countries to cope with temporary foreign exchange liquidity problems. SDRs are granted in proportion to the IMF quotas with the approval of 85% in voting power of the participants. If an IMF country has insufficient amount of a currency needed for payment of a trade deficit, it can transfer SDRs instead.

Answer (C) is incorrect because SDRs are created by an agreement among governments; they are entries in the accounts of the IMF that have been decreed by participating countries to be acceptable in lieu of other currencies. They are not used by commercial banks.

Answer (D) is incorrect because any country belonging to the IMF can use SDRs.

[44] Source: CMA 0688 1-25

Answer (A) is incorrect because currencies do not have an inherent tendency to depreciate or appreciate.

Answer (B) is correct. Exchange rates "float" when they are set by supply and demand, not by agreement among countries. In a managed float, central banks buy and sell currencies at their discretion to avoid erratic fluctuations in the foreign currency market. The objective of such transactions is to "manage" the level at which a particular currency sells in the open market. For instance, if there is an oversupply of a country's currency on the foreign currency market, the central bank will purchase that currency to support the market.

Answer (C) is incorrect because currencies no longer have to be supported by gold.

Answer (D) is incorrect because central banks, not private business people, manage the quantity of

currency on the market.

[45] Source: CMA 0688 1-29

Answer (A) is incorrect because this restriction increases the price to U.S. consumers. The reduced supply may not be matched by reduced demand.

Answer (B) is incorrect because the government will not directly benefit from the restriction on auto imports from Japan. The restriction benefits U.S. manufacturers and helps equalize the balance of payments.

Answer (C) is correct. A voluntary agreement to restrict exports to the United States forestalls possible trade restrictions. Such an agreement is more likely if Japan's domestic profit margins are sufficiently high that its dependence on exports to the U.S. has been reduced.

Answer (D) is incorrect because Japan will want to export more expensive cars to the U.S. The higher-priced cars will have higher margins of profit.

[46] Source: CMA 0688 1-30

Answer (A) is incorrect because although most exchanges quote the price of gold in U.S. dollars, the dollar's value is not linked to that of gold.

Answer (B) is incorrect because floating exchange rates have existed since about 1973. Tying currency values to a gold standard, in effect, fixes exchange rates.

Answer (C) is correct. Gold has no special role in the modern international monetary system. The present system is based upon managed floating currency exchange rates. Consequently, gold is treated as a commodity, the price of which depends upon supply and demand.

Answer (D) is incorrect because the only reserves of the IMF are international currencies.

[47] Source: CMA 1288 1-12

Answer (A) is incorrect because Taiwan does not have an absolute advantage with respect to either product.

Answer (B) is correct. Given two countries and two products, each country will always have a comparative advantage with respect to one of the products. Each country may or may not have an absolute advantage. In this case, the U.S. has an absolute advantage with respect to both products because with one unit of resources it can produce more soybeans (12 to 6) and more chips (1,800 to 1,500) than Taiwan. Comparatively, the U.S. has an advantage with respect to soybeans. Comparative advantage is measured by determining which product can be manufactured more cheaply (for the lower opportunity cost) in comparison to the other country. For 1 ton of soybeans, the U.S. opportunity cost is 150 units (1,800/12) of chips. The opportunity cost

20

Page 21: CMA Part 1B International Business Environment)

for Taiwan is 250 units (1,500/6) of chips.

Answer (C) is incorrect because the U.S. has the comparative advantage in the production of soybeans, and Taiwan has the comparative advantage in the production of chips. Taiwan's opportunity cost for the production of one unit of chips is .004 ton (6/1,500) of soybeans. U.S. chip production has an opportunity cost of .0067 ton (12/1,800) of soybeans.

Answer (D) is incorrect because the U.S. has the comparative advantage in the production of soybeans, and Taiwan has the comparative advantage in the production of chips. Taiwan's opportunity cost for the production of one unit of chips is .004 ton (6/1,500) of soybeans. U.S. chip production has an opportunity cost of .0067 ton (12/1,800) of soybeans.

[48] Source: CMA 1288 1-13

Answer (A) is incorrect because both countries will benefit from free trade. Specialization results in the optimal total output.

Answer (B) is incorrect because the U.S. will specialize in the production of soybeans.

Answer (C) is incorrect because Taiwan will produce chips and export them to the U.S.

Answer (D) is correct. Countries should specialize in the production and export of products for which they have a comparative advantage. They should import those products for which they do not have a comparative advantage. Thus, Taiwan will specialize in the production of chips, and the U.S. will specialize in the production of soybeans.

[49] Source: CMA 1288 1-14

Answer (A) is incorrect because 1 ton of soybeans for 250 chips is the opportunity cost for Taiwan.

Answer (B) is correct. Taiwan's opportunity cost for 1 ton of soybeans is 250 units of chips. The U.S.'s opportunity cost for 1 ton of soybeans is 150 units of chips (1800/12). Thus, the lowest price that the U.S. should charge is 150 units of chips per ton of soybeans. Moreover, Taiwan should not pay more than 250 units of chips for imported soybeans because they could be made domestically for that price. Therefore, the trading price must be between 150 and 250 units of chips per ton of soybeans if trade is to be advantageous.

Answer (C) is incorrect because the lowest price the U.S. should charge is 150 units of chips per ton of soybeans and the maximum price that Taiwan would be willing to pay is 250 units of chips per ton of soybeans.

Answer (D) is incorrect because the lowest price that the U.S. would charge is 150 units of chips per ton of soybeans. Additionally, Taiwan would not be willing to pay more than 250 units of chips per ton of soybeans.

[50] Source: CMA 1288 1-15

Answer (A) is incorrect because the trade account still has a deficit despite the cheaper dollar. Imports continue to exceed exports because patterns of consumption are slow to change.

Answer (B) is incorrect because the capital account benefits from the cheaper dollar. Foreigners can buy more dollars with fewer yen, marks, etc. Moreover, foreign capital inflow increases because of the federal government's budget deficits. Hence, the U.S. experiences a net capital inflow.

Answer (C) is incorrect because the fall in the dollar has a positive effect on the nation's trade deficit. Exports increase and imports decrease.

Answer (D) is correct. A decline in the value of the dollar relative to other currencies lowers the price of U.S. goods to foreign consumers. Thus, exporters of domestically produced goods benefit. A low value of the dollar also decreases imports by making foreign goods more expensive.

[51] Source: CMA 1288 1-16

Answer (A) is correct. The price difference of three yen is a difference of 2 ス% (3 ・120) for 90 days. Annualizing this difference (multiplying by 4) produces an annual premium of 10%. Because the 90-day price of the dollar is higher than the current price, it is said that the dollar is at a premium on the 90-day forward market.

Answer (B) is incorrect because 2.5% is the premium for 90 days.

Answer (C) is incorrect because the effect is a 10% premium, not discount.

Answer (D) is incorrect because the 90-day effect is a 2.5% or 0.025 premium.

[52] Source: CMA 1288 1-17

Answer (A) is incorrect because 735 pounds results from considering only the effect of the 5% decline in the pound.

Answer (B) is incorrect because 721 pounds results from considering only the effect of the 3% U.S. inflation rate.

Answer (C) is correct. Two factors are to be considered: (1) the 3% inflation rate and (2) the 5% decline in the pound. Considering the inflation rate first, 3% should be added to the immediate delivery price to arrive at a future price of 721 pounds. However, to allow for the foreign currency exchange-rate risk, an additional 5% should be added to the inflation-adjusted price. The bid price should be 757 pounds [721 pounds + (5% x 721 pounds)].

Answer (D) is incorrect because the effects of both

21

Page 22: CMA Part 1B International Business Environment)

the 3% U.S. inflation rate and the 5% decline in the pound need to be added to the initial 700 pounds.

[53] Source: CMA 1288 1-18

Answer (A) is incorrect because inflation affects exchange rates by diminishing a currency's purchasing power.

Answer (B) is correct. Since Italy has experienced the greater inflation, its currency should depreciate in relation to Canada's. For example, if Canada trades 100 units of a product to Italy for a preinflation price of $100 (the domestic price in Canada), and Italy pays with 10,000 units of an Italian product that sells domestically for 10,000 preinflation lira, the exchange rate without regard to inflation is 100 lira per $1 (10,000 lira/$100). Allowing for the inflation, the 100 units of the Canadian product would sell for $105. The 10,000 units of the Italian product would sell for 11,000 lira. Thus, the new exchange rate will be 104.76 lira per $1 (11,000 lira/$105), and the price of the Canadian dollar will increase by 4.76% (rounded to 5%).

Answer (C) is incorrect because the Canadian currency will appreciate relative to Italy's since Canadian inflation was lower.

Answer (D) is incorrect because the Canadian currency will appreciate relative to Italy's since Canadian inflation was lower.

[54] Source: CMA 1288 1-19

Answer (A) is incorrect because the current account includes exports and imports of services as well as goods.

Answer (B) is incorrect because the trade balance concerns goods only.

Answer (C) is correct. The balance of payments accounts include all international payments made by one nation to another, including capital movements, imports, exports, and unilateral transfers. The net of exports and imports is the current account balance. The current account balance does not include capital transactions. The capital account reflects movements of financial capital (investments).

Answer (D) is incorrect because the various components are never in balance. In reality, the important consideration is the total, and that is more of a long-run than an annual problem.

[55] Source: CMA 1288 1-20

Answer (A) is incorrect because the gold standard was abandoned by the IMF in 1973.

Answer (B) is incorrect because currencies have floating values.

Answer (C) is correct. Special drawing rights (SDRs)

were created by the International Monetary Fund (IMF) to enable countries to cope with temporary foreign exchange liquidity problems. SDRs are granted in proportion to the IMF quotas with the approval of 85% in voting power of the participants. If an IMF country has insufficient amount of a currency needed for payment of a trade deficit, it can transfer SDRs instead.

Answer (D) is incorrect because SDRs are reserves created by the IMF for use by nations in settling international accounts.

[56] Source: CMA 1288 1-21

Answer (A) is incorrect because the current account has a deficit, not a surplus, of Pta 7,000.

Answer (B) is incorrect because investment by foreigners should be deducted from, not added to, domestic investment in foreign countries. Furthermore, interest payments on foreign loans affect the current, not the capital account, and the capital account has a trade deficit, not a surplus.

Answer (C) is incorrect because investment by foreigners should be deducted from, not added to, domestic investment in foreign countries.

Answer (D) is correct. The balance of trade is the balance of imports and exports of goods. The current account also considers trade in services (none for this economy), unilateral transfers (e.g., gifts), and investment receipts and payments. It does not include capital transactions. Thus, investments by foreigners in the domestic economy and investments made in foreign countries will not be included in the current account. These transactions will be reflected in the capital account. The capital account will therefore have a deficit of 4,900 (6,300 - 1,400) pesetas, since investment in foreign countries is greater than investment by foreigners. The current account will have a deficit of 7,000 pesetas (15,760 - 20,300 + 1,240 - 3,700). This question assumes that the balance of trade is synonymous with the current account balance. But these terms are often differentiated.

[57] Source: CMA 0691 1-16

Answer (A) is correct. Import tariffs and quotas cause the prices of goods to increase. Tariffs directly increase the prices paid by consumers. Quotas decrease supply and therefore indirectly raise the price. At the same time, resources will shift from relatively efficient export industries to less efficient protected industries. Price will rise as resources are misallocated to high-cost producers.

Answer (B) is incorrect because prices will increase.

Answer (C) is incorrect because the volume of trade will decline as less efficient domestic producers remain in or enter the market. Total worldwide real output will also decline.

Answer (D) is incorrect because employment in the protected industries will increase in the short run, but

22

Page 23: CMA Part 1B International Business Environment)

total employment may not be affected.

[58] Source: CMA 0691 1-18

Answer (A) is incorrect because imports are debited.

Answer (B) is incorrect because expenditures of American tourists abroad are treated as imports and are therefore debited.

Answer (C) is incorrect because earnings of foreigners represent outflows of foreign exchange and are debited.

Answer (D) is correct. Basically, exports of goods and services are credited to the U.S. balance of payments accounts and imports are debited. Similarly, capital movements may be debited or credited. For example, transfers of capital from foreigners to Americans, such as loans, are credits in the accounts. In effect, these loans constitute exports of debt instruments and increase the supply of foreign exchange available for the U.S.

[59] Source: CMA 1293 1-25

Answer (A) is incorrect because the capital account includes capital movements only; the direction of capital movements is influenced by the prevailing interest rates in each nation.

Answer (B) is incorrect because official reserves are assets held by central banks and are not necessarily related to current transactions.

Answer (C) is incorrect because this is a nonsense answer.

Answer (D) is correct. The balance of payments represents all international payments made by one nation to another, including those for imports, exports, investments, unilateral transfers such as pensions and gifts, and capital movements. The principal accounts are the current account and the capital account. The current account includes a net trade balance in goods, net investment receipts or payments, net receipts or payments for services, and the balance of unilateral transfers.

[60] Source: CMA 1293 1-26

Answer (A) is incorrect because, in the 1980s, high real interest rates made investments in the U.S. more attractive. Thus, demand for U.S. dollars increased.

Answer (B) is incorrect because a large demand for dollars drives up the price of dollars relative to other currencies.

Answer (C) is correct. Many factors influence the value of a country's currency on the international market. These factors include interest rate differentials, inflation differentials, balance of trade, balance of payments, and stability of governments. However, a demand by Americans for more foreign goods would drive down the price of the dollar because of the resulting increased demand for foreign

currencies.

Answer (D) is incorrect because the stability of the U.S. government and its currency made the dollar a secure store of value in the eyes of many foreigners; these foreigners therefore acquired dollars to hold as a safeguard against inflation in their own countries.

[61] Source: CMA 1293 1-27

Answer (A) is incorrect because the balance of trade is the difference between imports and exports of goods.

Answer (B) is incorrect because, beyond some level of output, returns diminish as more and more units of an input are added to the production process.

Answer (C) is incorrect because relative competition is not a term relevant to international economics.

Answer (D) is correct. The relevant concept is comparative advantage, which compares the costs of inputs within a single country. In contrast, the concept of absolute advantage compares the costs of inputs between countries. It is possible that a country might have an absolute advantage with respect to every product, but comparative advantage is different from absolute advantage. A particular nation can have a comparative advantage even though it does not have an absolute advantage. For example, assume that Country A can produce Item X for $100 and Item Y for $200 and that Country B can produce Item X for $50 and Item Y for $150. B has an absolute advantage in the production of both products; however, B has a comparative advantage in producing Item X (50 ・100, or 50% of the A cost, compared with 150 ・200, or 75% of the A cost for Item Y). A has a comparative advantage in producing Item Y (200 ・150, or 133% of the B cost, versus 100 ・50, or 200%for Item X). A nation will benefit by exporting goods in which it has a comparative advantage and importing goods in which it does not have a comparative advantage. Total output will be maximized when each nation specializes in the products in which it has the greatest comparative advantage or the least comparative disadvantage.

[62] Source: CIA 1193 IV-68

Answer (A) is incorrect because military self-sufficiency is an argument for increasing trade barriers.

Answer (B) is incorrect because diversification for stability is an argument for increasing trade barriers by promoting industrial diversification and less dependence on other nations for certain products.

Answer (C) is incorrect because protection of infant industries is an argument for increasing trade barriers for the purpose of allowing new domestic industries to establish themselves.

Answer (D) is correct. The general effect of free trade would be to maximize world output because resources in each country would be deployed most

23

Page 24: CMA Part 1B International Business Environment)

efficiently according to the principle of comparative advantage. Comparative advantage means that a country can produce a greater output of certain goods for a given level of input than other goods. Thus, that country should specialize in and export the goods it can produce most efficiently. Total world output will increase in these circumstances.

[63] Source: CIA 0594 IV-58

Answer (A) is incorrect because Eurodollar Deposits are U.S. dollar deposits in banks outside of the U.S.

Answer (B) is incorrect because Eurodollar Deposits are outside the direct control of the U.S. monetary authorities.

Answer (C) is correct. Eurodollars are U.S. dollars on deposit in a foreign bank. These deposits are created when a check is drawn on a dollar deposit in a U.S. bank and then deposited in a bank outside the U.S. This amount is then available for lending by the foreign bank to its customers. However, the depositors still hold claims denominated in dollars. Because Eurodollars are outside the direct control of the U.S. monetary authorities, U.S. banking regulations with respect to reserves, insurance, interest ceilings, etc., do not apply. The absence of these costs means that Eurodollar deposit rates tend to be higher, not lower, than domestic U.S. rates on equivalent instruments.

Answer (D) is incorrect because interest rates on Eurodollar Deposits are tied to the London Interbank Offer Rate (LIBOR).

[64] Source: CIA 0592 IV-70

Answer (A) is correct. In the short run, a central bank's sale of the currency increases the supply and reduces the price of the currency. In the long run, given the current system of managed floating exchange rates, changes in rates should reflect changes in economic conditions. In other words, exchange rates should float. But central banks are expected to manage the float by buying and selling currencies to counteract the disruptive effects on rates of such temporary factors as speculation.

Answer (B) is incorrect because buying domestic currency in the foreign exchange market would raise the world-wide value of the domestic currency.

Answer (C) is incorrect because selling foreign currency would raise the world-wide value of the domestic currency with respect to that foreign currency.

Answer (D) is incorrect because a central bank decision to increase domestic interest rates would make the domestic currency attractive to foreign investors, and raise the value of the domestic currency.

[65] Source: CIA 1190 IV-58

Answer (A) is correct. The returns on the stock are

presumably paid in marks. Hence, the change in the value of the mark relative to the dollar does not affect the German company's return. However, the weakening of the mark reduces the number of dollars it will buy, and the U.S. company's return in dollars is correspondingly reduced.

Answer (B) is incorrect because the return to the U.S. company is adversely affected by the exchange rate movement.

Answer (C) is incorrect because the return to the U.S. company is adversely affected by the exchange rate movement.

Answer (D) is incorrect because the return to the U.S. company was directly and adversely affected by the exchange rate movement in the short-run, but the return to the German company was not.

[66] Source: CMA 0694 1-4

Answer (A) is correct. Exchange rates fluctuate depending upon the demand for each country's currency. If a country raises its interest rates, its currency will appreciate. The demand for investment at the higher interest rates will shift the demand curve for the currency to the right. The reverse holds true for a decrease in interest rates.

Answer (B) is incorrect because the currency should increase in relative value when interest rates in the country rise sharply. More investors will want to earn the higher rates of interest.

Answer (C) is incorrect because the currency should increase in relative value when interest rates in the country rise sharply. More investors will want to earn the higher rates of interest.

Answer (D) is incorrect because the currency should increase in relative value when interest rates in the country rise sharply. More investors will want to earn the higher rates of interest.

[67] Source: CMA 0694 1-5

Answer (A) is incorrect because world output will be maximized if each country specializes in those products in which they have a comparative advantage.

Answer (B) is incorrect because comparative advantage, not absolute advantage, determines the products in which a country should specialize.

Answer (C) is correct. The concept of comparative advantage compares costs within a single country. It is the ability of one nation to produce a product at a relatively lower opportunity cost (benefits forgone) than another nation. Absolute advantage compares the costs of inputs between countries. One country might have an absolute advantage with respect to every product. However, total output will be maximized when each nation specializes in the products in which it has the lowest opportunity costs, that is, a comparative advantage.

24

Page 25: CMA Part 1B International Business Environment)

Answer (D) is incorrect because, given a reciprocal absolute advantage, a corresponding comparative advantage will exist.

[68] Source: CMA 0695 1-23

Answer (A) is incorrect because domestic producers of export goods are not benefitted. Indeed, they may be harmed by retaliatory tariffs.

Answer (B) is correct. Despite the advantages of free trade, nations often levy tariffs to discourage the importation of certain products. A tariff is a tax on imports intended to protect a domestic producer from foreign competition. For instance, a tariff on imported autos benefits U. S. auto manufacturers because it is an additional cost imposed on U.S. consumers of such products. The disadvantages of the tariff are that it may protect an inefficient domestic producer and increase prices paid by domestic consumers.

Answer (C) is incorrect because domestic consumers must pay higher prices for imported goods.

Answer (D) is incorrect because the foreign producers will be forced to bear an additional cost.

[69] Source: CMA 0695 1-24

Answer (A) is incorrect because an increase in imports drives down the value of the nation's currency.

Answer (B) is incorrect because a high rate of inflation devalues a nation's currency.

Answer (C) is correct. Assuming that exchange rates are allowed to fluctuate freely, a nation's currency will appreciate if the demand for it is constant or increasing while supply is decreasing. For example, if the nation decreases its imports relative to exports, less of its currency will be used to buy foreign currencies for import transactions and more of its currency will be demanded for export transactions. Thus, the supply of the nation's currency available in foreign currency markets decreases. If the demand for the currency increases or does not change, the result is an increase in (appreciation of) the value of the currency.

Answer (D) is incorrect because lower interest rates relative to those in other countries discourage foreign investment, decreases demand for the nation's

currency, and reduces its value.

[70] Source: CIA 0595 IV-59

Answer (A) is incorrect because Eurodollar borrowers tend to pay lower, not higher, rates. Borrowers and depositors can both receive more favorable rates because, with its lower costs, the Eurodollar market can offer smaller spreads between borrowing and lending rates.

Answer (B) is incorrect because U.S. dollars are on

deposit in both cases.

Answer (C) is incorrect because Eurodollar deposits tend to be for larger, not smaller, amounts. Furthermore, smaller deposits tend to earn lower, not higher, rates than larger deposits.

Answer (D) is correct. Eurodollars are U.S. dollars deposited in banks outside the U.S. Because it is outside the direct control of the U.S. monetary authorities, the Eurodollar market has lower costs. For example, U.S. reserve requirements and FDIC premium payments do not apply in this market. A lower cost market can offer depositors higher interest rates.

[71] Source: CIA 1195 IV-66

Answer (A) is incorrect because Eurobonds are not always denominated in Eurodollars, which are U.S. dollars deposited outside the U.S.

Answer (B) is correct. Eurobonds are, by definition, always sold in some country other than the one in whose currency the bond issue is denominated. Their advantage is that they are customarily less stringently regulated than most other bonds. Hence, transaction costs are lower.

Answer (C) is incorrect because foreign bonds are denominated in the currency of the country in which they are sold.

Answer (D) is incorrect because Eurobonds are usually issued not as registered bonds but as bearer bonds, so names and nationalities of the investors are not recorded.

[72] Source: CIA 0596 IV-69

Answer (A) is incorrect because the total balance of payments surplus or deficit, which is the sum of the current and capital account balances, is what determines changes in official reserves and pressures for exchange rates to change.

Answer (B) is incorrect because the total balance of payments surplus or deficit, which is the sum of the current and capital account balances, is what determines changes in official reserves and pressures for exchange rates to change.

Answer (C) is incorrect because, given an appreciation of the currency, the short-run effects are likely to be a balance of payments surplus and growing official reserves.

Answer (D) is correct. In a freely floating exchange-rate system, exchange rates automatically adjust so as to eliminate balance of payments surpluses or deficits. For example, if U.S. demand for country X's currency increases, the result is a U.S. deficit at the existing exchange rate because demand now exceeds the supply of X's currency at that rate. However, the system of floating exchange rates allows the change in the relative strength of the currencies to be reflected in their exchange rate. The appreciation of X's currency against the U.S. dollar,

25

Page 26: CMA Part 1B International Business Environment)

that is, the increase in the amount of U.S. dollars exchangeable for a unit of X's currency, makes U.S. products cheaper to buyers in country X. Furthermore, X's products are more expensive to U.S. buyers. Consequently, U.S. imports will fall, U.S. exports will rise, and the balance of payments deficit will decrease.

[73] Source: CIA 1195 IV-67

Answer (A) is incorrect because, if incomes in country 1 rise, the result will be a tendency for it to devalue relative to the currency of country 2.

Answer (B) is incorrect because, if incomes in country 1 fall, consumers in country 1 will reduce their imports. The resulting decrease in the supply of currency 1 will result in a tendency for it to appreciate relative to the currency of country 2.

Answer (C) is correct. If incomes in country 1 rise, consumers in country 1 will increase their imports from country 2. The resulting increase in the supply of currency 1 will result in a tendency for it to depreciate relative to the currency of country 2.

Answer (D) is incorrect because, if incomes in country 1 remain constant, the currency of country 1 will not tend to appreciate or depreciate relative to the currency of country 2.

[74] Source: CIA 1196 IV-64

Answer (A) is incorrect because a forward market sale of foreign currency is appropriate to hedge a receivable denominated in a foreign currency.

Answer (B) is incorrect because waiting to buy the currency in 60 days does not eliminate the risk of an adverse exchange-rate movement.

Answer (C) is correct. The company can arrange to purchase the foreign currency today rather than in 60 days by buying the currency in the forward market. This hedging transaction will eliminate the exchange-rate risk associated with the trade payable.

Answer (D) is incorrect because this strategy would be comparable to a future sale of the foreign currency at a rate known today, which would not provide the currency needed to pay the invoice. However, the opposite strategy would be an effective money market hedge. If the company converted domestic currency to foreign currency in the spot market today and invested in a foreign bank deposit or treasury bill, it could then use the proceeds from the foreign investment to pay the invoice in 60 days.

[75] Source: CIA 1196 IV-73

Answer (A) is correct. If the exchange rate changes from $1 being worth 5 francs to $1 being worth 5.5 francs, the U.S. dollar has appreciated by 10% [(5.5 - 5) ・5].

Answer (B) is incorrect because the U.S. dollar has appreciated in value. It will now purchase more

francs.

Answer (C) is incorrect because the franc is depreciating, not appreciating.

Answer (D) is incorrect because the franc has depreciated by 9.09%.

[76] Source: CIA 1196 IV-74

Answer (A) is correct. If the real rates of interest are equal, the country with the higher nominal interest rate is expected to experience a higher rate of inflation. A higher rate of inflation is associated with a devaluing currency, so the currency of the country with the higher nominal interest rate will likely be selling at a forward discount.

Answer (B) is incorrect because the currency of country A will be selling at a forward discount.

Answer (C) is incorrect because the spot relationship between the two currencies cannot be determined from the information given.

Answer (D) is incorrect because the spot relationship between the two currencies cannot be determined from the information given.

[77] Source: CIA 1195 IV-68

Answer (A) is incorrect because domestic producers will sell more, and domestic consumers will consume less, as a result of the imposition of a protective tariff.

Answer (B) is correct. Domestic producers are not subject to the tariff and will therefore have a price advantage over their foreign competitors. However, absent such competition, the domestic price of the item will be higher. Domestic producers will sell more at a higher price, and domestic consumers will consume less following the price increase.

Answer (C) is incorrect because domestic producers will sell more, and domestic consumers will consume less, as a result of the imposition of a protective tariff.

Answer (D) is incorrect because domestic producers will sell more, and domestic consumers will consume less, as a result of the imposition of a protective tariff.

[78] Source: CIA 0595 IV-64

Answer (A) is incorrect because a tariff is a tax levied by a foreign government against certain imported products. A firm exporting to that country must accept lower profits, absorbing the tariff, or increase selling prices in the foreign country to compensate. The tariff reduces profitability or competitiveness in the foreign market but does not exclude the firm from exporting to that country.

Answer (B) is incorrect because a quota is a limit on the amount of goods that an importing country will accept in certain product categories. The effect of a quota is to restrict the quantity the firm can export to that country but not to exclude the firm from selling in

26

Page 27: CMA Part 1B International Business Environment)

that market. The effect on revenues and profitability depends on market conditions in that country.

Answer (C) is correct. An embargo is a total ban on some kinds of imports. It is an extreme form of the import quota. Embargoes have the effect of totally excluding the exporting firm from selling in that country and are the most restrictive type of import/export law.

Answer (D) is incorrect because exchange controls limit the amount of foreign exchange that can be transacted or the exchange rate against other currencies. These controls limit the ability of a firm selling in the country to repatriate its export earnings but do not exclude the firm from selling in that market.

[79] Source: CIA 0594 IV-64

Answer (A) is incorrect because licensing requirements limit exports, e.g., of militarily sensitive technology.

Answer (B) is correct. Tariffs are excise taxes on imported goods imposed either to generate revenue or protect domestic producers. Thus, consumption taxes on imported goods are tariffs.

Answer (C) is incorrect because unreasonable standards pertaining to product quality and safety are nontariff trade barriers.

Answer (D) is incorrect because domestic content rules require that a portion of an imported good be made in the importing country.

[80] Source: CIA 0594 IV-65

Answer (A) is incorrect because a protective tariff can only increase the domestic price of the imported item.

Answer (B) is correct. A protective tariff adds to the purchase price of imported goods. If an imported good's sales price is higher than a comparable, less expensive domestic good, consumers will purchase the domestic good. Thus, the direct effect of imposing a protective tariff on an imported good is lower domestic consumption.

Answer (C) is incorrect because, as the imported item's domestic price increases, demand for domestic goods will increase. Thus, domestic production will increase, not decrease.

Answer (D) is incorrect because, as the imported item's domestic price increases, demand for the item decreases. Lower sales revenues will result.

[81] Source: Publisher

Answer (A) is incorrect because prices paid by the German buyer will increase. It must adjust payments in marks upward for both U. S. inflation and German

monetary depreciation.

Answer (B) is incorrect because prices paid by the German buyer will increase. It must adjust payments in marks upward for both U. S. inflation and German monetary depreciation.

Answer (C) is incorrect because prices would increase by 3% simply because of inflation. This answer disregards the effect of the currency depreciation.

Answer (D) is correct. Assuming the original exchange rate is $1 to 2 marks and that U.S. inflation is 3%, the cost in marks to purchase what once cost $1 will now be 2.06 marks (2 x 1.03). However, if the mark also depreciates by 12%, that is, if the mark is expected to be worth 88% of its current value, the exchange rate (before inflation) will be $1 to 2.2727 (2 ・.88) marks. At this rate, 2.3409 marks (2.2727 x 1.03) will be required to purchase $1. Costs in marks will therefore increase by just over 17% (.3409/2.00).

[82] Source: Publisher

Answer (A) is incorrect because Iceland has the absolute advantage with respect to cheese.

Answer (B) is incorrect because Holland has the

absolute advantage in soda production.

Answer (C) is incorrect because Holland has the absolute advantage with respect to soda.

Answer (D) is correct. Comparative advantage compares costs of multiple products within a single country. For example, Holland has a comparative advantage in soda because it can produce soda more cheaply than cheese. Absolute advantage compares the costs of inputs between countries. In Holland, the price of 10 million pounds of cheese is 30 million cases of soda, or a 1-to-3 ratio. In Iceland, the price of 10 million pounds of cheese is 10 million cases of soda, or a 1-to-1 ratio. Thus, Iceland has an absolute advantage in cheese because it can produce cheese

more cheaply than Holland.

[83] Source: Publisher

Answer (A) is incorrect because it is based on a reversal of the beginning and ending spot rates.

Answer (B) is incorrect because it is based only on the change in spot rates--not the 10% return.

Answer (C) is incorrect because the nominal return must be adjusted for the change in spot prices.

Answer (D) is correct. The $100,000 divided by .65 resulted in $153,846.15 being invested in terms of Canadian dollars. A 10% return would increase that amount to $169,230.76 in Canadian dollars. Converting $169,230.76 in Canadian dollars to U.S. dollars (by multiplying by .7) results in $118,461.53 in U.S. dollars. Dividing the $118,461.53 by the

27

Page 28: CMA Part 1B International Business Environment)

$100,000 original investment results in a return of 18.46%.

[84] Source: CIA 1196 IV-78

Answer (A) is incorrect because the inclusion of the tariff increases the domestic price.

Answer (B) is incorrect because foreign sales in the domestic market decline from ad to bc.

Answer (C) is incorrect because domestic production increases from Oa to Ob.

Answer (D) is correct. Without the tariff, domestic production is determined by the intersection of the Pw line with the domestic supply curve at the quantity Oa. Domestic production increases from Oa to Ob as a result of the introduction of the tariff. Supply intersects the Pt line at a higher price and at a greater domestic quantity, Ob.

[85] Source: Publisher

Answer (A) is correct. The U.S. must give up 8 beef sides for each 4 units of lard, or a ratio of 2 beef sides to 1 unit of lard. In Canada, the same ratio is 12 beef sides for 3 units of lard, or 4 beef sides to 1 unit of lard. Thus, the exchange rate will be somewhere between 2 and 4 beef sides to 1 unit of lard.

Answer (B) is incorrect because the exchange rate for lard will be between 1/4 and 1/2 unit to 1 beef side.

Answer (C) is incorrect because the exchange rate for lard will be between 1/4 and 1/2 unit to 1 beef side.

Answer (D) is incorrect because the exchange rate for lard will be between 1/4 and 1/2 unit to 1 beef side.

[86] Source: Publisher

Answer (A) is correct. Canada has a comparative advantage with respect to beef and could produce 60 units, enough to cover exactly the current production in both countries (24 + 36). The U.S. could produce 20 units of lard, which is 6 units greater than the current demand of 14 (8 + 6).

Answer (B) is incorrect because the total increase is only 6 units.

Answer (C) is incorrect because there is no increase in beef sides.

Answer (D) is incorrect because there is no increase in beef sides.

[87] Source: Publisher

Answer (A) is incorrect because there is an increment of 12 over the current production of 24.

Answer (B) is incorrect because 24 is the current production without specialization.

Answer (C) is correct. If the U.S. produces 20 units of lard and keeps only 8 units, 12 units are being traded. At a 3-to-1 trade rate, the U.S. will receive 36 beef sides for its 12 units of lard.

Answer (D) is incorrect because only 36 sides will be available at a 3-to-1 ratio.

[88] Source: Publisher

Answer (A) is incorrect because you have to give up only 1/3 of a tractor to get 1 car.

Answer (B) is incorrect because the cost of 1 tractor is 3 cars.

Answer (C) is correct. In Bulgaria, 300 cars have to be given up to get 100 tractors, or 3 cars for 1 tractor. Thus, the cost of 1 tractor is 3 cars. Inversely, the cost of 1 car is 1/3 of a tractor.

Answer (D) is incorrect because the cost of 3 tractors is 9 cars.

[89] Source: Publisher

Answer (A) is incorrect because this statement is true.

Answer (B) is incorrect because this statement is true.

Answer (C) is correct. Andorra should not specialize in the production of tractors because it does not have a comparative advantage in tractors. Total output is maximized when each country specializes in the products in which it has the lower opportunity cost (or comparative advantage). The cost of a tractor in Andorra is 5 cars (1,000 ・200), whereas the cost of a tractor in Bulgaria is 3 cars (300 ・100). Thus, Bulgaria has the comparative advantage with tractors.

Answer (D) is incorrect because this statement is true.

[90] Source: Publisher

Answer (A) is incorrect because Andorra would not be willing to pay more than 5 cars for each tractor, since cars can be produced domestically for that price.

Answer (B) is incorrect because Andorra would not be willing to pay more than 5 cars for each tractor, since cars can be produced domestically for that price.

Answer (C) is correct. The cost of a tractor in Andorra is 5 cars (1,000 ・200), whereas the cost of a tractor in Bulgaria is 3 cars (300 ・100). Thus, the terms of trade will be somewhere between 3 cars and 5 cars for each tractor.

28

Page 29: CMA Part 1B International Business Environment)

Answer (D) is incorrect because Bulgaria, the country with the comparative advantage in tractors, will require at least 3 cars for each tractor.

[91] Source: Publisher

Answer (A) is correct. Under specialization, each country produces those products for which it has a comparative advantage. Thus, Bulgaria would produce 500 tractors, while Andorra would produce 4,000 cars. Under alternative C for Bulgaria and alternative B for Andorra, the total production of cars would be 3,900 (900 for Bulgaria plus 3,000 for Andorra). Thus, the 4,000 under specialization is an increase of 100 cars. Bulgaria would produce 200 tractors, while Andorra would produce 200, a total of 400 tractors without specialization - again, an increase of 100.

Answer (B) is incorrect because specialization will result in production gains of only 100 units of each product.

Answer (C) is incorrect because specialization will result in production gains of only 100 units of each product.

Answer (D) is incorrect because specialization will result in production gains of only 100 units of each product.

[92] Source: Publisher

Answer (A) is incorrect because $0.20 is based on an inversion of the numerator and denominator in the calculation.

Answer (B) is correct. At a 1-for-9 rate, the price in U.S. dollars is $5, calculated by dividing 45 pesos by 9.

Answer (C) is incorrect because $45 is the price in pesos, not dollars.

Answer (D) is incorrect because $405 is based on multiplying 45 and 9.

[93] Source: Publisher

Answer (A) is incorrect because $.20 is the exchange rate for the dollar, not the pound.

Answer (B) is correct. Dividing $100,000 by 」20,000 produces an exchange rate of $5 to the pound.

Answer (C) is incorrect because the exchange rate is $5 to the pound.

Answer (D) is incorrect because the exchange rate is $5 to the pound.

[94] Source: Publisher

Answer (A) is incorrect because the franc will depreciate rather than appreciate.

Answer (B) is correct. Currently, the franc sells for $.90. In a year, the price will drop to $.85. At $.90, a person can purchase 100 francs for $90. Next year, the same 100 francs will cost $85, meaning the franc will be less valuable (in terms of dollars). Thus, the value of the franc will depreciate against the dollar. The amount of the decline of $5, divided by the original price of $90, will result in a 5.56% depreciation.

Answer (C) is incorrect because the franc will depreciate rather than appreciate.

Answer (D) is incorrect because it uses .85 in the denominator instead of .90.

29